Difference between revisions of "1999 AHSME Problems/Problem 28"
Isabelchen (talk | contribs) m (→Solution 2) |
|||
(3 intermediate revisions by the same user not shown) | |||
Line 9: | Line 9: | ||
<math> \mathrm{(A) \ }3 \qquad \mathrm{(B) \ }4 \qquad \mathrm{(C) \ }5 \qquad \mathrm{(D) \ }6 \qquad \mathrm{(E) \ }7 </math> | <math> \mathrm{(A) \ }3 \qquad \mathrm{(B) \ }4 \qquad \mathrm{(C) \ }5 \qquad \mathrm{(D) \ }6 \qquad \mathrm{(E) \ }7 </math> | ||
− | == Solution == | + | == Solution 1== |
Clearly, we can ignore the possibility that some <math>x_i</math> are zero, as adding/removing such variables does not change the truth value of any condition, nor does it change the value of the sum of cubes. Thus we'll only consider <math>x_i\in\{-1,1,2\}</math>. | Clearly, we can ignore the possibility that some <math>x_i</math> are zero, as adding/removing such variables does not change the truth value of any condition, nor does it change the value of the sum of cubes. Thus we'll only consider <math>x_i\in\{-1,1,2\}</math>. | ||
Line 34: | Line 34: | ||
The maximum is achieved for | The maximum is achieved for | ||
<cmath>(\underbrace{-1,\dots,-1}_{21},1,1,\underbrace{2,\dots,2}_{19})</cmath> | <cmath>(\underbrace{-1,\dots,-1}_{21},1,1,\underbrace{2,\dots,2}_{19})</cmath> | ||
+ | |||
+ | ==Solution 2== | ||
+ | |||
+ | As said in Solution 1, <math>x_i=0</math> can be ignored, and only <math>x_i = -1, 1, 2</math> need to be considered. | ||
+ | |||
+ | ===Minimum=== | ||
+ | |||
+ | To minimize <math>x_1^3 + \cdots + x_n^3</math>, there are no <math>2</math>s and maximize the number of <math>-1</math>s. | ||
+ | |||
+ | Therefore, the number of <math>1</math>s are <math>19 + \frac{99-19}{2} = 59</math>, the number of <math>-1</math>s are <math>\frac{99-19}{2} = 40</math>. | ||
+ | |||
+ | <math>x_1^3 + \cdots + x_n^3 = 59 - 40 = 19</math> | ||
+ | |||
+ | ===Maximum=== | ||
+ | |||
+ | Let <math>a =</math> number of <math>2</math>s, <math>b =</math> number of <math>1</math>s, <math>c =</math> number of <math>-1</math>s | ||
+ | |||
+ | <cmath>4a+b+c=99</cmath> | ||
+ | <cmath>2a+b-c=19</cmath> | ||
+ | |||
+ | Multiplying the second equation by <math>2</math> gives <math>4a+2b-2c=38</math>. By subtracting this equation from the first equation we get <math>3c-b=61</math>, <math>3c=61+b</math>. As we need to minimize the value of <math>c</math>, <math>c = 21</math>, <math>b = 2</math>, <math>a = 19</math> | ||
+ | |||
+ | <math>x_1^3 + \cdots + x_n^3 = 8 \cdot 19 +2 - 21 = 133</math> | ||
+ | |||
+ | Therefore, <math>\frac Mm = \frac{133}{19}=\boxed{\textbf{(E) } 7}</math>. | ||
+ | |||
+ | ~[https://artofproblemsolving.com/wiki/index.php/User:Isabelchen isabelchen] | ||
== See also == | == See also == | ||
{{AHSME box|year=1999|num-b=27|num-a=29}} | {{AHSME box|year=1999|num-b=27|num-a=29}} | ||
{{MAA Notice}} | {{MAA Notice}} |
Latest revision as of 11:25, 3 October 2023
Problem
Let be a sequence of integers such that (i) for (ii) ; and (iii) . Let and be the minimal and maximal possible values of , respectively. Then
Solution 1
Clearly, we can ignore the possibility that some are zero, as adding/removing such variables does not change the truth value of any condition, nor does it change the value of the sum of cubes. Thus we'll only consider .
Also, order of the does not matter, so we are only interested in the counts of the variables of each type. Let of the be equal to , equal to , and equal to .
The conditions (ii) and (iii) simplify to:
(ii)
(iii)
and we want to find the maximum and minimum of over all non-negative solutions of the above two equations.
Subtracting twice (ii) from (iii) we get . By entering that into one of the two equations and simplifying we get .
Thus all the solutions of our system of equations have the form .
As all three variables must be non-negative integers, we have and .
For of the form the expression we are maximizing/minimizing simplifies to . Clearly, the maximum is achieved for and the minimum for . Their values are and , and therefore .
Note
The minimum is achieved for The maximum is achieved for
Solution 2
As said in Solution 1, can be ignored, and only need to be considered.
Minimum
To minimize , there are no s and maximize the number of s.
Therefore, the number of s are , the number of s are .
Maximum
Let number of s, number of s, number of s
Multiplying the second equation by gives . By subtracting this equation from the first equation we get , . As we need to minimize the value of , , ,
Therefore, .
See also
1999 AHSME (Problems • Answer Key • Resources) | ||
Preceded by Problem 27 |
Followed by Problem 29 | |
1 • 2 • 3 • 4 • 5 • 6 • 7 • 8 • 9 • 10 • 11 • 12 • 13 • 14 • 15 • 16 • 17 • 18 • 19 • 20 • 21 • 22 • 23 • 24 • 25 • 26 • 27 • 28 • 29 • 30 | ||
All AHSME Problems and Solutions |
The problems on this page are copyrighted by the Mathematical Association of America's American Mathematics Competitions.